Last visit was: 25 Apr 2024, 01:56 It is currently 25 Apr 2024, 01:56

Close
GMAT Club Daily Prep
Thank you for using the timer - this advanced tool can estimate your performance and suggest more practice questions. We have subscribed you to Daily Prep Questions via email.

Customized
for You

we will pick new questions that match your level based on your Timer History

Track
Your Progress

every week, we’ll send you an estimated GMAT score based on your performance

Practice
Pays

we will pick new questions that match your level based on your Timer History
Not interested in getting valuable practice questions and articles delivered to your email? No problem, unsubscribe here.
Close
Request Expert Reply
Confirm Cancel
SORT BY:
Kudos
Tags:
Show Tags
Hide Tags
Senior Manager
Senior Manager
Joined: 07 Dec 2017
Posts: 325
Own Kudos [?]: 1663 [15]
Given Kudos: 348
GMAT 1: 650 Q50 V28
GMAT 2: 720 Q49 V40
Send PM
Current Student
Joined: 31 Jul 2017
Status:He came. He saw. He conquered. -- Going to Business School -- Corruptus in Extremis
Posts: 1734
Own Kudos [?]: 5738 [4]
Given Kudos: 3054
Location: United States (MA)
Concentration: Finance, Economics
Send PM
Manager
Manager
Joined: 19 Jan 2018
Posts: 178
Own Kudos [?]: 122 [1]
Given Kudos: 79
Send PM
GMAT Club Verbal Expert
Joined: 13 Aug 2009
Status: GMAT/GRE/LSAT tutors
Posts: 6920
Own Kudos [?]: 63659 [1]
Given Kudos: 1773
Location: United States (CO)
GMAT 1: 780 Q51 V46
GMAT 2: 800 Q51 V51
GRE 1: Q170 V170

GRE 2: Q170 V170
Send PM
Re: Joshi is clearly letting campaign contributions influence his vote [#permalink]
1
Kudos
Expert Reply
ashutosh_73 wrote:
Hi Experts,

I find (A), (D), (E) equally tempting. Can't find errors to eliminate them, please clarify.

Stimulus breakdown:

P1: Re-election received more financial support from property developers

+

P2: voting recd. favors the interest of property developers

=====================================

Conclusion:letting campaign contri influence his vote in city C

Here's (D):

Quote:
presumes that one thing is the cause of another when it could easily be an effect of it

Joshi is on the city council right now. His past voting record favors the interests of property developers. Property developers are supporting his re-election campaign with contributions.

The author concludes from this that the campaign contributions are CAUSING his pro-property votes. But what if he was already pro-property, and THAT caused property developers to support him with contributions? The argument totally misses this -- it assumes that the contributions caused the votes, but it could easily be the other way around.

(D) is looking good.

Here's (A):

Quote:
takes for granted that because certain events occurred sequentially, the earlier events caused the later events

The sequence of events is:

  • Joshi is on the council.
  • Joshi votes pro-property developer.
  • Property developers support Joshi's re-election campaign.

The author assumes that the LATER event (#3) caused the EARLIER event (#2). That's the opposite of what (A) says, so (A) isn't good grounds to criticize the argument.

Eliminate (A).

Here's (E):

Quote:
has a conclusion that is simply a restatement of one of the argument's stated premises

The author concludes that "Joshi is clearly letting campaign contributions influence his vote in city council." In other words, the voting record is CAUSED by the campaign contributions.

Then, as evidence for this, the author cites Joshi's voting record, and separately the campaign contributions. The conclusion does something really different by saying that one of these things causes the other.

So, we can't criticize the argument by saying that the conclusion simply restates the premises.

(E) is out, and (D) is the correct answer.

I hope that helps!
Retired Moderator
Joined: 23 Sep 2015
Posts: 1267
Own Kudos [?]: 5650 [0]
Given Kudos: 416
Send PM
Re: Joshi is clearly letting campaign contributions influence his vote [#permalink]
AshutoshB wrote:
Joshi is clearly letting campaign contributions influence his vote in city council. His campaign for re-election has received more financial support from property developers than any other city councilors has. And more than any other councilor's, his voting record favors the interests of property developers.

The reasoning in the argument is most vulnerable to criticism on the grounds that the argument


(A) takes for granted that because certain events occurred sequentially, the earlier events caused the later events

(B) confuses one thing's being necessary for another to occur with its being sufficient to make it occur

(C) makes a moral judgment when only a factual judgment can be justified

(D) presumes that one thing is the cause of another when it could easily be an effect of it

(E) has a conclusion that is simply a restatement of one of the argument's stated premises



LSAT


This one is easy. D should be the answer. its a weaken question. it may be possible that he is in the campaign because of a support from those developers.
Manager
Manager
Joined: 17 Jan 2016
Status:As cheeks from my insta feed say: soon...
Posts: 68
Own Kudos [?]: 20 [0]
Given Kudos: 144
Send PM
Re: Joshi is clearly letting campaign contributions influence his vote [#permalink]
AshutoshB wrote:
Joshi is clearly letting campaign contributions influence his vote in city council. His campaign for re-election has received more financial support from property developers than any other city councilors has. And more than any other councilor's, his voting record favors the interests of property developers.

The reasoning in the argument is most vulnerable to criticism on the grounds that the argument


(A) takes for granted that because certain events occurred sequentially, the earlier events caused the later events

(B) confuses one thing's being necessary for another to occur with its being sufficient to make it occur

(C) makes a moral judgment when only a factual judgment can be justified

(D) presumes that one thing is the cause of another when it could easily be an effect of it

(E) has a conclusion that is simply a restatement of one of the argument's stated premises



LSAT


nightblade354, bro, i am begging you
GMAT Club Bot
Re: Joshi is clearly letting campaign contributions influence his vote [#permalink]
Moderators:
GMAT Club Verbal Expert
6920 posts
GMAT Club Verbal Expert
238 posts
CR Forum Moderator
832 posts

Powered by phpBB © phpBB Group | Emoji artwork provided by EmojiOne